Last year a record

This topic has expert replies
Senior | Next Rank: 100 Posts
Posts: 99
Joined: Sun Mar 13, 2011 12:42 pm

Last year a record

by factor26 » Sun Feb 12, 2012 5:29 pm
Last year a record number of new manufacturing jobs
were created. Will this year bring another record? Well,
a new manufacturing job is created either within an
existing company or by the start-up of a new company.
Within existing fi rms, new jobs have been created this
year at well below last year's record pace. At the same
time, there is considerable evidence that the number
of new companies starting up will be no higher this
year than it was last year, and surely the new
companies starting up this year will create no
more jobs per company than did last year's
start-ups
. Clearly, it can be concluded that the
number of new jobs created this year will fall
short of last year's record.

In the argument given, the two portions in boldface
play which of the following roles?
(A) The fi rst is a prediction that, if accurate, would
provide support for the main conclusion of the
argument; the second is that main conclusion.
(B) The fi rst is a prediction that, if accurate, would
provide support for the main conclusion of the
argument; the second is a conclusion drawn in
order to support that main conclusion.
(C) The fi rst is an objection that the argument
rejects; the second is the main conclusion of the
argument.
(D) The fi rst is an objection that the argument
rejects; the second presents a conclusion that
could be drawn if that objection were allowed to
stand.
(E) The fi rst is a claim that has been advanced in
support of a position that the argument
opposes; the second is a claim advanced in
support of the main conclusion of the argument

OG answer is A. I see that A and B are nearly identical but contain slightly different working toward the end of the sentence. Can someone please explain in further detail, answer choices A and B? Thanks!
Last edited by factor26 on Mon Feb 13, 2012 5:04 pm, edited 1 time in total.

User avatar
Legendary Member
Posts: 1239
Joined: Tue Apr 26, 2011 6:25 am
Thanked: 233 times
Followed by:26 members
GMAT Score:680

by sam2304 » Sun Feb 12, 2012 7:17 pm
Where is the bold part ? ? Please use proper formatting while posting questions. :)
Getting defeated is just a temporary notion, giving it up is what makes it permanent.
https://gmatandbeyond.blogspot.in/

Senior | Next Rank: 100 Posts
Posts: 99
Joined: Sun Mar 13, 2011 12:42 pm

by factor26 » Mon Feb 13, 2012 5:06 pm
Sorry Sam - edited the original with the boldface statements noted - thanks for the heads up!

Newbie | Next Rank: 10 Posts
Posts: 3
Joined: Wed Jan 25, 2012 2:48 am

by Pavan M » Thu Feb 23, 2012 9:55 pm
Clearly we can make out that A & B are the contenders. B says that the second bold face is the point which supports the main conclusion which is wrong. Because, it's very clear that the second bold face is the main conclusion. Hence the answer should be A.

Legendary Member
Posts: 2789
Joined: Tue Jul 26, 2011 12:19 am
Location: Chennai, India
Thanked: 206 times
Followed by:43 members
GMAT Score:640

by GmatKiss » Sat Feb 25, 2012 6:16 am
Bad post :( :( , why can't it be aligned properly before posting.

Do we need to read 50 more line to answer a CR! Strange and embarrassing post!

Senior | Next Rank: 100 Posts
Posts: 99
Joined: Sun Mar 13, 2011 12:42 pm

by factor26 » Sat Feb 25, 2012 8:52 am
@ GMATkiss - if you have a problem with my post I suggest you move onto a new post instead of posting such a classless comment.